Last visit was: 08 May 2024, 20:55 It is currently 08 May 2024, 20:55

Close
GMAT Club Daily Prep
Thank you for using the timer - this advanced tool can estimate your performance and suggest more practice questions. We have subscribed you to Daily Prep Questions via email.

Customized
for You

we will pick new questions that match your level based on your Timer History

Track
Your Progress

every week, we’ll send you an estimated GMAT score based on your performance

Practice
Pays

we will pick new questions that match your level based on your Timer History
Not interested in getting valuable practice questions and articles delivered to your email? No problem, unsubscribe here.
Close
Request Expert Reply
Confirm Cancel
User avatar
Intern
Intern
Joined: 01 Sep 2009
Posts: 19
Own Kudos [?]: 643 [0]
Given Kudos: 0
Send PM
User avatar
Manager
Manager
Joined: 23 May 2008
Posts: 224
Own Kudos [?]: 1696 [0]
Given Kudos: 14
Send PM
avatar
Manager
Manager
Joined: 29 Jul 2009
Posts: 178
Own Kudos [?]: 1487 [0]
Given Kudos: 9
 Q50  V40
Send PM
User avatar
Manager
Manager
Joined: 23 May 2008
Posts: 224
Own Kudos [?]: 1696 [0]
Given Kudos: 14
Send PM
Re: GMATPrep CR [#permalink]
mikeCoolBoy wrote:
The author's conclusion is the following:

It had been assumed that people lack sufficient free time to maintain current television-viewing levels while spending increasing amounts of free time on the computer. The author conclusion is that this is false because time spent using the computer has increased and time spent watching TV hasn't changed. So the author conclusion is that people have enough time to use the computer and watch TV

Now you have to apply the variance test among the answer choices, looking for different effects in the conclusion. You have to look for an answer choice that if true weakens/strengthens the argument and if false strengthens/weakens the argument.

A. Whether a large majority of the survey respondents reported watching television regularly

Whether this is true/false does not matter since we are interested in the amount of time spent, but not in how often people watch TV.

B. Whether the amount of time spent watching television is declining among people who report that they rarely or never use computers

The conclusion covers people who use computers and people who do not. So if you reduce the scope to people who rarely or never use computers, you can't use the information to evaluate the argument.

C. Whether the type of television programs a person watches tends to change as the amount of time spent per week using computers increases

Again you have to keep the scope of the argument, amount of time spent on watching television vs amount of time spent on using computers. You are not interested in whether the types of television programs changes.

D. Whether a large majority of the computer owners in the survey reported spending increasing amounts of time per week using computers

Notice the shift in the scope since this question only talks about computer owners and the conclusion includes all the respondents. You can't use this information to evaluate the argument.

E. Whether the survey respondents’ reports of time spent using computers included time spent using computers at work

if this is true, it weakens the author's conclusion because the increased in time spent using computers includes time at work, so perhaps people do not have enough free time to watch TV and use the computer.

If this is false, it strengthens the author's conclusion because people have used computers outside work (free time) and the time spent watching TV hasn't changed. Therefore people must have enough free time to use computers and to watch TV.



yes i think you are right.....even i realized it later that the argument is not concerned with the types of television programme.......So E is the best choice...
RC & DI Moderator
Joined: 02 Aug 2009
Status:Math and DI Expert
Posts: 11221
Own Kudos [?]: 32356 [0]
Given Kudos: 301
Send PM
Re: GMATPrep CR [#permalink]
Expert Reply
ans is E:-
the answer to the ques would tell us whether computer usage is limited to only free time or it includes working hours...
if yes the argument weakens as we r talking only of free hours in argument
User avatar
Intern
Intern
Joined: 22 Mar 2009
Posts: 49
Own Kudos [?]: 138 [0]
Given Kudos: 7
Concentration: Business
 Q49  V29 GMAT 2: 700  Q48  V36
GPA: 3.5
WE 1: 3
Send PM
Re: GMATPrep CR [#permalink]
The question we have to ask is " how did the computer watching-hours" increased even though the "tv watching hours" remained the same???


Option E definilty asnwers HOW..

E is my answer..


" brothers " pls post the right answer
User avatar
Manager
Manager
Joined: 16 Apr 2009
Posts: 137
Own Kudos [?]: 464 [0]
Given Kudos: 14
Send PM
Re: GMATPrep CR [#permalink]
I will also go with E. Mikecoolboy has already provided good reasoning.



Archived Topic
Hi there,
This topic has been closed and archived due to inactivity or violation of community quality standards. No more replies are possible here.
Where to now? Join ongoing discussions on thousands of quality questions in our Critical Reasoning (CR) Forum
Still interested in this question? Check out the "Best Topics" block above for a better discussion on this exact question, as well as several more related questions.
Thank you for understanding, and happy exploring!
GMAT Club Bot
Re: GMATPrep CR [#permalink]
Moderators:
GMAT Club Verbal Expert
6922 posts
GMAT Club Verbal Expert
238 posts

Powered by phpBB © phpBB Group | Emoji artwork provided by EmojiOne